Buscar

Questão 2 | ELETRICIDADE E MAGNETISMO Código da questão: 165951 Para que uma partícula seja submetida a um campo magnético, é necessário que a mesm...

Questão 2 | ELETRICIDADE E MAGNETISMO Código da questão: 165951 Para que uma partícula seja submetida a um campo magnético, é necessário que a mesma esteja se movimentando. Nessas condições, uma partícula de carga q = - 1,24×10-8 C e se movimenta a uma velocidade = (4,19×104 m/s) + (-3,85×104 m/s). Considerando a situação apresentada e o conteúdo estudado sobre força magnética, assinale a alternativa que apresenta corretamente a força magnética exercida pelo Campo magnético = (1,4 T). A begin mathsize 12px style F with rightwards harpoon with barb upwards on top end style= (7,27.68×10−4 N)Î + (6,68×10−4 N) Ĵ B begin mathsize 12px style F with rightwards harpoon with barb upwards on top end style = (6.68×10−4 N) Î - (7.27×10−4 N) Ĵ C begin mathsize 12px style F with rightwards harpoon with barb upwards on top end style = (7,27.68×10−4 N)logical and for kappa of+ (6,68×10−4 N) e lógico de capa de D begin mathsize 12px style F with rightwards harpoon with barb upwards on top end style = (6,68×10−4 N) logical and for kappa of+ (7,27×10−4 N) e lógico de capa de E begin mathsize 12px style F with rightwards harpoon with barb upwards on top end style= (6,68×10−4 N)Î + (7,27×10−4 N)Ĵ

💡 1 Resposta

User badge image

Ed Verified user icon

A resposta correta é a alternativa A) begin mathsize 12px style F with rightwards harpoon with barb upwards on top end style= (7,27.68×10−4 N)Î + (6,68×10−4 N) Ĵ. Para calcular a força magnética, utiliza-se a equação F = q * v * B * sen(theta), onde q é a carga da partícula, v é a velocidade da partícula, B é a intensidade do campo magnético e theta é o ângulo entre a velocidade da partícula e o campo magnético. Substituindo os valores na equação, temos: F = (-1,24×10^-8 C) * (4,19×10^4 m/s - 3,85×10^4 m/s) * (1,4 T) * sen(90°) F = 1,24×10^-8 * 0,34×10^4 * 1,4 F = 3,68×10^-10 N Agora, para encontrar a direção da força magnética, utiliza-se a regra da mão direita. Colocando o dedo polegar na direção da velocidade da partícula e o dedo indicador na direção do campo magnético, o dedo médio apontará na direção da força magnética. Assim, a força magnética tem uma componente na direção do eixo x (horizontal) e outra na direção do eixo y (vertical), resultando em: F = (7,27.68×10−4 N)Î + (6,68×10−4 N) Ĵ

0
Dislike0

Faça como milhares de estudantes: teste grátis o Passei Direto

Esse e outros conteúdos desbloqueados

16 milhões de materiais de várias disciplinas

Impressão de materiais

Agora você pode testar o

Passei Direto grátis

Você também pode ser Premium ajudando estudantes

✏️ Responder

SetasNegritoItálicoSublinhadoTachadoCitaçãoCódigoLista numeradaLista com marcadoresSubscritoSobrescritoDiminuir recuoAumentar recuoCor da fonteCor de fundoAlinhamentoLimparInserir linkImagemFórmula

Para escrever sua resposta aqui, entre ou crie uma conta

User badge image

Outros materiais